Difference between revisions of "2015 AMC 8 Problems/Problem 25"

(Solution 1)
(Video Solutions)
 
(105 intermediate revisions by 47 users not shown)
Line 1: Line 1:
 +
== Problem ==
 +
 +
 
One-inch squares are cut from the corners of this 5 inch square.  What is the area in square inches of the largest square that can be fitted into the remaining space?
 
One-inch squares are cut from the corners of this 5 inch square.  What is the area in square inches of the largest square that can be fitted into the remaining space?
 
<math> \mathrm{(A) \ } 9\qquad \mathrm{(B) \ } 12\frac{1}{2}\qquad \mathrm{(C) \ } 15\qquad \mathrm{(D) \ } 15\frac{1}{2}\qquad \mathrm{(E) \ } 17</math>
 
  
 
<asy>
 
<asy>
 +
size(75);
 
draw((0,0)--(0,5)--(5,5)--(5,0)--cycle);
 
draw((0,0)--(0,5)--(5,5)--(5,0)--cycle);
 
filldraw((0,4)--(1,4)--(1,5)--(0,5)--cycle, gray);
 
filldraw((0,4)--(1,4)--(1,5)--(0,5)--cycle, gray);
Line 11: Line 13:
 
</asy>
 
</asy>
  
===Solution 1===
+
<math>\textbf{(A)  } 9\qquad \textbf{(B) } 12\frac{1}{2}\qquad \textbf{(C)  } 15\qquad \textbf{(D)  } 15\frac{1}{2}\qquad \textbf{(E)  } 17</math>
We draw a diagram as shown.
+
 
<asy>
+
 
draw((0,0)--(0,5)--(5,5)--(5,0)--cycle);
+
==Video Solutions==
filldraw((0,4)--(1,4)--(1,5)--(0,5)--cycle, gray);
+
 
filldraw((0,0)--(1,0)--(1,1)--(0,1)--cycle, gray);
+
 
filldraw((4,0)--(4,1)--(5,1)--(5,0)--cycle, gray);
+
 
filldraw((4,4)--(4,5)--(5,5)--(5,4)--cycle, gray);
+
 
path arc = arc((2.5,4),1.5,0,90);
+
https://youtu.be/rTljQV79PCY
pair P = intersectionpoint(arc,(0,5)--(5,5));
+
 
pair Pp=rotate(90,(2.5,2.5))*P, Ppp = rotate(90,(2.5,2.5))*Pp, Pppp=rotate(90,(2.5,2.5))*Ppp;
+
~savannahsolver
draw(P--Pp--Ppp--Pppp--cycle);
+
 
</asy>
+
https://youtu.be/51K3uCzntWs?t=3358
Let us focus on the big triangles taking up the rest of the space. The triangles on top of the unit square between the inscribed square, are similiar to the 4 big triangles by AA.  Let the height of a big triangle be <math>x</math> then <math>\dfrac{x}{x-1}=\dfrac{5-x}{1}</math>.
+
 
<cmath>x=-x^2+6x-5</cmath>
+
~ pi_is_3.14
<cmath>x^2-5x+5=0</cmath>
 
<cmath>x=\dfrac{5\pm \sqrt{(-5)^2-(4)(1)(5)}}{2}</cmath>
 
<cmath>x=\dfrac{5\pm \sqrt{5}}{2}</cmath>
 
Which means <math>x=\dfrac{5-\sqrt{5}}{2}</math>
 
This means the area of each triangle is <math>\dfrac{5-\sqrt{5}}{2}*(5-\dfrac{5-\sqrt{5}}{2})*\dfrac{1}{2}=\dfrac{5}{2}</math>
 
This the area of the square is <math>25-(4*\dfrac{5}{2})=\boxed{C,~15}</math>
 
  
===Solution 2=== 
+
==See Also==
  
We draw a diagram as shown:
 
<asy>
 
pair Q,R,S,T;
 
Q=(1.381966,0);
 
R=(5,1.381966);
 
S=(3.618034,5);
 
T=(0,3.618034);
 
draw((0,0)--(0,5)--(5,5)--(5,0)--cycle);
 
filldraw((0,4)--(1,4)--(1,5)--(0,5)--cycle, gray);
 
filldraw((0,0)--(1,0)--(1,1)--(0,1)--cycle, gray);
 
filldraw((4,0)--(4,1)--(5,1)--(5,0)--cycle, gray);
 
filldraw((4,4)--(4,5)--(5,5)--(5,4)--cycle, gray);
 
draw(Q--R--S--T--cycle);
 
draw((1,1)--(4,1)--(4,4)--(1,4)--cycle,dashed);
 
</asy>
 
  
We wish to find the area of the larger triangle.  The area of the larger square is composed of the smaller square and the four triangles.  The triangles have base <math>3</math> and height <math>1</math>, so the combined area of the four triangles is <math>4 \cdot \frac 32=6</math>.  The area of the smaller square is <math>9</math>.  We add these to see that the area of the large square is <math>9+6=\boxed{\mathrm{(C) \ } 15}</math>.
+
{{AMC8 box|year=2015|num-b=24|after=Last Problem}}
 +
{{MAA Notice}}

Latest revision as of 10:02, 23 July 2024

Problem

One-inch squares are cut from the corners of this 5 inch square. What is the area in square inches of the largest square that can be fitted into the remaining space?

[asy] size(75); draw((0,0)--(0,5)--(5,5)--(5,0)--cycle); filldraw((0,4)--(1,4)--(1,5)--(0,5)--cycle, gray); filldraw((0,0)--(1,0)--(1,1)--(0,1)--cycle, gray); filldraw((4,0)--(4,1)--(5,1)--(5,0)--cycle, gray); filldraw((4,4)--(4,5)--(5,5)--(5,4)--cycle, gray); [/asy]

$\textbf{(A)  } 9\qquad \textbf{(B)  } 12\frac{1}{2}\qquad \textbf{(C)  } 15\qquad \textbf{(D)  } 15\frac{1}{2}\qquad \textbf{(E)  } 17$


Video Solutions

https://youtu.be/rTljQV79PCY

~savannahsolver

https://youtu.be/51K3uCzntWs?t=3358

~ pi_is_3.14

See Also

2015 AMC 8 (ProblemsAnswer KeyResources)
Preceded by
Problem 24
Followed by
Last Problem
1 2 3 4 5 6 7 8 9 10 11 12 13 14 15 16 17 18 19 20 21 22 23 24 25
All AJHSME/AMC 8 Problems and Solutions

The problems on this page are copyrighted by the Mathematical Association of America's American Mathematics Competitions. AMC logo.png